Question

In: Economics

Q3 . A cost function for a beef production business is give below; TC = 100...

Q3 . A cost function for a beef production business is give below;

TC = 100 + 3y – 0.2y2 + 0.04y3

a. From the above cost function, derive the exact equation for;

i. Marginal Cost (MC)

ii. Average Cost (AC)

b. Using the above cost function, calculate;

i. Total Fixed Cost

ii. Total Variable Cost

iii. Total Marginal when Y = 4

iv. Level of output Y when Average Variable Cost is at minimum.

Solutions

Expert Solution

ai) Marginal Cost : The Marginal Cost (MC) at Y items is the cost of producing the next item. It's formula is given as the following-

aii) Average Cost : The Average Cost (AC) for Y items is the total cost divided by Y. It's formula is given as the following :

b i) Total Fixed Cost : The Fixed Cost (FC) is the amount of money you have to spend regardless of how many items you produce. It is the component of the fixed cost which doesn't depend on the output produced, thus here :

b ii) Total Variable Cost: The Total Variable Cost (TVC) for Y items is the amount of money you spend to actually produce them. Thus, it depends on the output produced.

TC= TFC + TVC

b iii) Marginal Cost (Y=4)

Substituting Y= 4 in the above equation, we get:


Related Solutions

Consider a monopolist whose total cost function is TC = Q3 – 30Q2 + 301Q, whose...
Consider a monopolist whose total cost function is TC = Q3 – 30Q2 + 301Q, whose inverse demand function is P = 1501 – 30Q where Q is output and P is the price. If the firm were to act as a single price monopolist, how much will it produce? What price it will charge? How large will be its profit? Calculate the firm’s Lerner Index of monopoly power. Now suppose this firm can practice perfect price discrimination. How much...
Q3. Assume that a competitive firm has the total cost function: TC=1q3-40q2+890q+1800 Suppose the price of...
Q3. Assume that a competitive firm has the total cost function: TC=1q3-40q2+890q+1800 Suppose the price of the firm's output (sold in integer units) is $600 per unit. Using tables (but not calculus) to find a solution, what is the total profit at the optimal output level? Please specify your answer as an integer.
A firm has the following production function Y=K0.25L0.25. Total cost (TC) is given by TC=wLL+wKK+ZC, where...
A firm has the following production function Y=K0.25L0.25. Total cost (TC) is given by TC=wLL+wKK+ZC, where wLand wK are prices of the two inputs L and K, and ZC are costs that the firm has to pay regardless of production volume as long as it is operative. a)Derive total cost as a function of output C(Y). Derive marginal cost MC and average cost AC. b)Assume that the firm is one of many identical ones operating on a perfectly competitive market...
A firm has the following production function Y=K0.25L0.25. Total cost (TC) is given by TC=wLL+wKK+ZC, where...
A firm has the following production function Y=K0.25L0.25. Total cost (TC) is given by TC=wLL+wKK+ZC, where wL and wK are prices of the two inputs L and K, and ZC are costs that the firm has to pay regardless of production volume as long as it is operative. Derive total cost as a function of output C(Y). Derive marginal cost MC and average cost AC. Assume that the firm is one of many identical ones operating on a perfectly competitive...
1. Suppose a firm has the following total cost function: TC = 100 + 4q^2 a....
1. Suppose a firm has the following total cost function: TC = 100 + 4q^2 a. What is the minimum price necessary for the firm to earn profit? You must explain your reasoning and process as to how your found the price you found. b. Below what price will the firm shut down in the short run?
4. You are given the following cost functions: TC= 100+ 60Q- 3Q2 + 0.1Q3 TC= 100+...
4. You are given the following cost functions: TC= 100+ 60Q- 3Q2 + 0.1Q3 TC= 100+ 60Q+ 3Q2 TC= 100+ 60Q a. Compute the average variable cost, average cost, and marginal cost for each function. Plot them on a graph. b. In each case, indicate the point at which diminishing returns occur. Also indicate the point of maximum cost efficiency (i.e., the point of minimum average cost). c. For each function, discuss the relationship between marginal cost and average variable...
Suppose a company's revenue function is given by R(q)=−q3+320q2R(q)=-q3+320q2 and its cost function is given by...
Suppose a company's revenue function is given by R(q)=−q3+320q2R(q)=-q3+320q2 and its cost function is given by C(q)=290+20qC(q)=290+20q, where qq is hundreds of units sold/produced, while R(q)R(q) and C(q)C(q) are in total dollars of revenue and cost, respectively. A) Find a simplified expression for the marginal profit function. (Be sure to use the proper variable in your answer.) MP(q)=MP(q)=      B) How many items (in hundreds) need to be sold to maximize profits? (Round your answer to two decimal places.)
A monopolist faces the demand function Q = 20 – 2P. Its cost function is TC(Q)...
A monopolist faces the demand function Q = 20 – 2P. Its cost function is TC(Q) =0.5Q. Solve for the monopolist’s profit-maximizing price and output and calculate its profit as well as the consumer surplus and deadweight loss.
The total cost (TC) and inverse demand equations for a monopolist are: TC=100+5Q^2 P=200?5Q a. What...
The total cost (TC) and inverse demand equations for a monopolist are: TC=100+5Q^2 P=200?5Q a. What is the profit-maximizing quantity? b. What is the profit-maximizing price? c. What is the monopolist's maximum profit? The demand equation for a product sold by a monopolist is Q=25?0.5P TC=225+5Q+0.25Q^2 a. Calculate the profit-maximizing price and quantity. b. What is the monopolist's profit?
A monopolist has total cost TC = Q2 + 10Q + 100 and marginalcost MC...
A monopolist has total cost TC = Q2 + 10Q + 100 and marginal cost MC = 2Q + 10. It faces demand Q = 130 - P (so its marginal revenue is MR = 130 - 2Q). Its profit-maximizing price is$50$75$100
ADVERTISEMENT
ADVERTISEMENT
ADVERTISEMENT